Answered step by step
Verified Expert Solution
Link Copied!

Question

1 Approved Answer

Two rms, Firm 1 and Firm 2, compete by simultaneously choosing prices. Both rms sell an identical product for which each of 100 consumers has

image text in transcribed
Two rms, Firm 1 and Firm 2, compete by simultaneously choosing prices. Both rms sell an identical product for which each of 100 consumers has a maximum willingness to pay of $40. Each consumer will buy at most 1 unit, and will buy it from whichever rm charges the lowest price. If both rms set the same price, they share the market equally. Costs are given by cf(q,) = 16q,. Because of government regulation, rms can only choose prices which are integer numbers, and they cannot price above $40. Answer the following: a) (0.25 point) If Firm 1 chooses p1 = 34, Firm 23 best response is to set what price? b) (0.25 point) If Firm 2 chooses the price determined in the previous question, Firm 'l's best response is to choose what price? c) (1 point) If Firm 1 chooses p1 = 4, Firm 2's best response is a range of prices. What is the lowest price in this range

Step by Step Solution

There are 3 Steps involved in it

Step: 1

blur-text-image

Get Instant Access with AI-Powered Solutions

See step-by-step solutions with expert insights and AI powered tools for academic success

Step: 2

blur-text-image

Step: 3

blur-text-image

Ace Your Homework with AI

Get the answers you need in no time with our AI-driven, step-by-step assistance

Get Started

Recommended Textbook for

Statistics For Engineers And Scientists

Authors: William Navidi

4th Edition

73401331, 978-0073401331

Students also viewed these Economics questions